Is x > y? (1) √x > y (2) x3 > y

This topic has expert replies
User avatar
Master | Next Rank: 500 Posts
Posts: 298
Joined: Sun Jun 03, 2012 6:42 am
Location: New delhi
Thanked: 10 times
Followed by:7 members
GMAT Score:590

Is x > y? (1) √x > y (2) x3 > y

by varun289 » Wed Sep 04, 2013 1:27 am
Is x > y? (1) √x > y (2) x3 > y

User avatar
Legendary Member
Posts: 643
Joined: Wed Aug 14, 2013 4:27 am
Thanked: 48 times
Followed by:7 members

by vinay1983 » Wed Sep 04, 2013 2:22 am
Effectively Statement 1

√x > y

√x can be 4, 5, 6 or 7 or x can be 2, 3.14 or 2.15 or something like that and can be greater than Y if, Y is negative or zero or any value less than the square root of x

No sure shot answer or information. One number can be great than another number only when we know the other number or th erelation between them

Statement 1 not sufficient

Statement 2

x^3 > y

so x can be assumed to be positive i.e 2 3 4 5 and if y can be assumed to positive and less than x.

But what can be the possibilities of x and y. Both can be integers, decimals or fractions.

So NOT Sufficient

Combining both, I could conclude that for some value of √x or x^3, y is lesser than √x.

So E for me.
You can, for example never foretell what any one man will do, but you can say with precision what an average number will be up to!

GMAT/MBA Expert

User avatar
Elite Legendary Member
Posts: 10392
Joined: Sun Jun 23, 2013 6:38 pm
Location: Palo Alto, CA
Thanked: 2867 times
Followed by:511 members
GMAT Score:800

by [email protected] » Wed Sep 04, 2013 11:54 pm
Hi vinay1983,

Rather than explain this entire question, I'm going to make a suggestion and have you go back and re-solve it:

A good way to prove what's going on is to TEST VALUES. You've presented a lot of ideas, but you've made a little mistake.

Go back and PROVE whatever you think. Pay attention to the information that you're given, TEST some specific numbers and track the results. You should be able to get this question correct.

GMAT assassins aren't born, they're made,
Rich
Last edited by [email protected] on Thu Sep 05, 2013 1:08 pm, edited 1 time in total.
Contact Rich at [email protected]
Image

User avatar
Legendary Member
Posts: 643
Joined: Wed Aug 14, 2013 4:27 am
Thanked: 48 times
Followed by:7 members

by vinay1983 » Thu Sep 05, 2013 1:08 am
[email protected] wrote:Hi vinay1983,

Rather than explain this entire question, I'm going to make a suggestion and have you go back and re-solve it:

A good way to prove what's going on is the TEST VALUES. You've presented a lot of ideas, but you've made a little mistake.

Go back and PROVE whatever you think. Pay attention to the information that you're given, TEST some specific numbers and track the results. You should be able to get this question correct.

GMAT assassins aren't born, they're made,
Rich

Rich, I am not able to follow what you are saying.


Stetement 1

√x > y

means x has to be more than y.Is this correct?

Statement 2

X^3 > y

means y has to be lesser than x?

Then I can Say answer would be D?

Sorry Rich!, could not kickstart my brain for this :(
You can, for example never foretell what any one man will do, but you can say with precision what an average number will be up to!

User avatar
GMAT Instructor
Posts: 15539
Joined: Tue May 25, 2010 12:04 pm
Location: New York, NY
Thanked: 13060 times
Followed by:1906 members
GMAT Score:790

by GMATGuruNY » Thu Sep 05, 2013 9:26 am
Private tutor exclusively for the GMAT and GRE, with over 20 years of experience.
Followed here and elsewhere by over 1900 test-takers.
I have worked with students based in the US, Australia, Taiwan, China, Tajikistan, Kuwait, Saudi Arabia -- a long list of countries.
My students have been admitted to HBS, CBS, Tuck, Yale, Stern, Fuqua -- a long list of top programs.

As a tutor, I don't simply teach you how I would approach problems.
I unlock the best way for YOU to solve problems.

For more information, please email me (Mitch Hunt) at [email protected].
Student Review #1
Student Review #2
Student Review #3

GMAT/MBA Expert

User avatar
Elite Legendary Member
Posts: 10392
Joined: Sun Jun 23, 2013 6:38 pm
Location: Palo Alto, CA
Thanked: 2867 times
Followed by:511 members
GMAT Score:800

by [email protected] » Thu Sep 05, 2013 1:13 pm
Hi vinay1983,

Compare what you wrote in your second post to what you wrote in your first post. You've written two completely different explanations with different answers.

I suggest that you pick SPECIFIC numbers, write them down and prove if the answer to the question changes or stays the same. You're attempting to "talk" your way past this question and you're making mistakes.

GMAT assassins aren't born, they're made,
Rich
Contact Rich at [email protected]
Image

User avatar
Legendary Member
Posts: 643
Joined: Wed Aug 14, 2013 4:27 am
Thanked: 48 times
Followed by:7 members

by vinay1983 » Thu Sep 05, 2013 6:57 pm
[email protected] wrote:Hi vinay1983,

Compare what you wrote in your second post to what you wrote in your first post. You've written two completely different explanations with different answers.

I suggest that you pick SPECIFIC numbers, write them down and prove if the answer to the question changes or stays the same. You're attempting to "talk" your way past this question and you're making mistakes.

GMAT assassins aren't born, they're made,
Rich
Hmmm,

Ok Combining both statements we have

√x > y and x^3 > y


X has to be positive and x has to be more than y, hence x is more than y!

Then C
You can, for example never foretell what any one man will do, but you can say with precision what an average number will be up to!

GMAT/MBA Expert

User avatar
GMAT Instructor
Posts: 3380
Joined: Mon Mar 03, 2008 1:20 am
Thanked: 2256 times
Followed by:1535 members
GMAT Score:800

by lunarpower » Fri Sep 13, 2013 11:39 pm
hey, i wrote this problem!
vinay1983 wrote:Ok Combining both statements we have

√x > y and x^3 > y


X has to be positive and x has to be more than y, hence x is more than y!

Then C
It's more complicated than that.

√x > y doesn't imply that x > y. If it did, then the answer would be (a), not (c).

Here's the thing:
The ORDER OF POWERS is different from "normal" for ...
... negatives
... numbers between 0 and 1.


Just try some numbers in these ranges, and you'll see.
If x < -1, then x^3 < x < x^2. (Plug in something like -2, and watch what falls out.)
If -1 < x < 0, then x < x^3 < x^2. (Plug in something like -1/2.)
If 0 < x < 1, then x^3 < x^2 < x < √x. (Plug in something like 1/2.)
If x > 1, then it's the "normal" order (√x < x < x^2 < x^3).
Note that there's no "√x" for negative values of x.

The reason why statement 1 isn't sufficient is the behavior of numbers between 0 and 1. For instance, if x = 1/4, then √x = 1/2, which is bigger.
So, for instance, if x = 1/4 and y = 1/3, then √x is greater than y, but x itself is less than y.

On the other hand, x is always between √x and x^3 (unless x = 0 or 1, in which case all three are the same). So that's why the two statements together are good enough.
Ron has been teaching various standardized tests for 20 years.

--

Pueden hacerle preguntas a Ron en castellano
Potete chiedere domande a Ron in italiano
On peut poser des questions à Ron en français
Voit esittää kysymyksiä Ron:lle myös suomeksi

--

Quand on se sent bien dans un vêtement, tout peut arriver. Un bon vêtement, c'est un passeport pour le bonheur.

Yves Saint-Laurent

--

Learn more about ron

User avatar
Legendary Member
Posts: 643
Joined: Wed Aug 14, 2013 4:27 am
Thanked: 48 times
Followed by:7 members

by vinay1983 » Fri Sep 13, 2013 11:54 pm
lunarpower wrote:hey, i wrote this problem!
vinay1983 wrote:Ok Combining both statements we have

√x > y and x^3 > y


X has to be positive and x has to be more than y, hence x is more than y!

Then C
It's more complicated than that.

√x > y doesn't imply that x > y. If it did, then the answer would be (a), not (c).

Here's the thing:
The ORDER OF POWERS is different from "normal" for ...
... negatives
... numbers between 0 and 1.


Just try some numbers in these ranges, and you'll see.
If x < -1, then x^3 < x < x^2. (Plug in something like -2, and watch what falls out.)
If -1 < x < 0, then x < x^3 < x^2. (Plug in something like -1/2.)
If 0 < x < 1, then x^3 < x^2 < x < √x. (Plug in something like 1/2.)
If x > 1, then it's the "normal" order (√x < x < x^2 < x^3).
Note that there's no "√x" for negative values of x.

The reason why statement 1 isn't sufficient is the behavior of numbers between 0 and 1. For instance, if x = 1/4, then √x = 1/2, which is bigger.
So, for instance, if x = 1/4 and y = 1/3, then √x is greater than y, but x itself is less than y.

On the other hand, x is always between √x and x^3 (unless x = 0 or 1, in which case all three are the same). So that's why the two statements together are good enough.
Ron!Am i glad to see you reply to this post!
Thanks for the clarification, but it would help me if you can simplify statement 1 for me a little more :(
You can, for example never foretell what any one man will do, but you can say with precision what an average number will be up to!

GMAT/MBA Expert

User avatar
GMAT Instructor
Posts: 3380
Joined: Mon Mar 03, 2008 1:20 am
Thanked: 2256 times
Followed by:1535 members
GMAT Score:800

by lunarpower » Sat Sep 14, 2013 12:06 am
vinay1983 wrote:it would help me if you can simplify statement 1 for me a little more :(
x = 1/4 and y = 1/3 satisfy statement 1. In that case, x < y.

x = 4 and y = 1 also satisfy statement 1. In that case, x > y.

Not sufficient.
Ron has been teaching various standardized tests for 20 years.

--

Pueden hacerle preguntas a Ron en castellano
Potete chiedere domande a Ron in italiano
On peut poser des questions à Ron en français
Voit esittää kysymyksiä Ron:lle myös suomeksi

--

Quand on se sent bien dans un vêtement, tout peut arriver. Un bon vêtement, c'est un passeport pour le bonheur.

Yves Saint-Laurent

--

Learn more about ron

User avatar
Legendary Member
Posts: 643
Joined: Wed Aug 14, 2013 4:27 am
Thanked: 48 times
Followed by:7 members

by vinay1983 » Sat Sep 14, 2013 1:21 am
lunarpower wrote:
vinay1983 wrote:it would help me if you can simplify statement 1 for me a little more :(
x = 1/4 and y = 1/3 satisfy statement 1. In that case, x < y.

x = 4 and y = 1 also satisfy statement 1. In that case, x > y.

Not sufficient.

Aha now i got it! I had erroneously tried to square the variables and also restricted the value of the variables to only positive integers. I did not consider fractions for them. My bad. I am struggling with plugging values. Thanks I will keep an eye on this.

Thanks a lot Ron!

Aha
You can, for example never foretell what any one man will do, but you can say with precision what an average number will be up to!

GMAT/MBA Expert

User avatar
GMAT Instructor
Posts: 3380
Joined: Mon Mar 03, 2008 1:20 am
Thanked: 2256 times
Followed by:1535 members
GMAT Score:800

by lunarpower » Sat Sep 14, 2013 4:08 am
vinay1983 wrote:and also restricted the value of the variables to only positive integers. I did not consider fractions for them. My bad. I am struggling with plugging values. Thanks I will keep an eye on this.
Whenever you see a problem involving the order of powers, numbers between 0 and 1 (and also negative numbers) should be on your radar.
Ron has been teaching various standardized tests for 20 years.

--

Pueden hacerle preguntas a Ron en castellano
Potete chiedere domande a Ron in italiano
On peut poser des questions à Ron en français
Voit esittää kysymyksiä Ron:lle myös suomeksi

--

Quand on se sent bien dans un vêtement, tout peut arriver. Un bon vêtement, c'est un passeport pour le bonheur.

Yves Saint-Laurent

--

Learn more about ron

Senior | Next Rank: 100 Posts
Posts: 68
Joined: Thu Jul 29, 2010 8:05 pm
Thanked: 3 times
Followed by:1 members

by ngalinh » Sat Sep 14, 2013 6:22 pm
An easy-to-make-mistake question but it looks simple at the beginning. Is there a way to create hard questions that it opens to many possibilities but still ties up at the end (sufficient), super gmatter? :)

GMAT/MBA Expert

User avatar
GMAT Instructor
Posts: 3380
Joined: Mon Mar 03, 2008 1:20 am
Thanked: 2256 times
Followed by:1535 members
GMAT Score:800

by lunarpower » Sun Sep 15, 2013 4:13 am
ngalinh wrote:An easy-to-make-mistake question but it looks simple at the beginning. Is there a way to create hard questions that it opens to many possibilities but still ties up at the end (sufficient), super gmatter? :)
Sorry, I don't know what this means.
Ron has been teaching various standardized tests for 20 years.

--

Pueden hacerle preguntas a Ron en castellano
Potete chiedere domande a Ron in italiano
On peut poser des questions à Ron en français
Voit esittää kysymyksiä Ron:lle myös suomeksi

--

Quand on se sent bien dans un vêtement, tout peut arriver. Un bon vêtement, c'est un passeport pour le bonheur.

Yves Saint-Laurent

--

Learn more about ron

Senior | Next Rank: 100 Posts
Posts: 68
Joined: Thu Jul 29, 2010 8:05 pm
Thanked: 3 times
Followed by:1 members

by ngalinh » Sun Sep 15, 2013 11:48 am
lunarpower wrote:
Sorry, I don't know what this means.
Oh, my mind wants to guess something again. It's always noisy. Never mind. I'll require it to stop.